consecutive prime gaps and explicit bound

The name of the pictureThe name of the pictureThe name of the pictureClash Royale CLAN TAG#URR8PPP











up vote
2
down vote

favorite
1












I am aware of the theorem that $p_n+1- p_n leq n^0.525$ which is true for all sufficiently large numbers due to Baker, but if i want to make the implicit "for all sufficiently large numbers" explicit, is it known that $p_n+1-p_n leq c n^alpha$ for all $n geq 1$ and for small $c$, lets say $c leq 2$ and $alpha leq 0.55$ ?



Any ref that can give me the explicit numbers or a way to construct them would be great.



Thank you, also i posted the question yesterday on MSE










share|cite|improve this question



























    up vote
    2
    down vote

    favorite
    1












    I am aware of the theorem that $p_n+1- p_n leq n^0.525$ which is true for all sufficiently large numbers due to Baker, but if i want to make the implicit "for all sufficiently large numbers" explicit, is it known that $p_n+1-p_n leq c n^alpha$ for all $n geq 1$ and for small $c$, lets say $c leq 2$ and $alpha leq 0.55$ ?



    Any ref that can give me the explicit numbers or a way to construct them would be great.



    Thank you, also i posted the question yesterday on MSE










    share|cite|improve this question

























      up vote
      2
      down vote

      favorite
      1









      up vote
      2
      down vote

      favorite
      1






      1





      I am aware of the theorem that $p_n+1- p_n leq n^0.525$ which is true for all sufficiently large numbers due to Baker, but if i want to make the implicit "for all sufficiently large numbers" explicit, is it known that $p_n+1-p_n leq c n^alpha$ for all $n geq 1$ and for small $c$, lets say $c leq 2$ and $alpha leq 0.55$ ?



      Any ref that can give me the explicit numbers or a way to construct them would be great.



      Thank you, also i posted the question yesterday on MSE










      share|cite|improve this question















      I am aware of the theorem that $p_n+1- p_n leq n^0.525$ which is true for all sufficiently large numbers due to Baker, but if i want to make the implicit "for all sufficiently large numbers" explicit, is it known that $p_n+1-p_n leq c n^alpha$ for all $n geq 1$ and for small $c$, lets say $c leq 2$ and $alpha leq 0.55$ ?



      Any ref that can give me the explicit numbers or a way to construct them would be great.



      Thank you, also i posted the question yesterday on MSE







      nt.number-theory reference-request analytic-number-theory prime-numbers prime-gaps






      share|cite|improve this question















      share|cite|improve this question













      share|cite|improve this question




      share|cite|improve this question








      edited 3 hours ago









      GH from MO

      56.4k5138214




      56.4k5138214










      asked 4 hours ago









      Ahmad

      33519




      33519




















          1 Answer
          1






          active

          oldest

          votes

















          up vote
          4
          down vote



          accepted










          The result you quote is due to Baker-Harman-Pintz (2000). I am not aware of any concrete effective version of this result, but if you increase the exponent $0.525$ to $2/3$, then such a variant is available by the work of Dudek. See also my response to this MO question.






          share|cite|improve this answer






















          • Thx, so is there $alpha <1$ such that $cleq 2$ and $n_0$ or $x_0$ is small say less than $10^10$ or $10^12 $ ?
            – Ahmad
            1 hour ago











          Your Answer




          StackExchange.ifUsing("editor", function ()
          return StackExchange.using("mathjaxEditing", function ()
          StackExchange.MarkdownEditor.creationCallbacks.add(function (editor, postfix)
          StackExchange.mathjaxEditing.prepareWmdForMathJax(editor, postfix, [["$", "$"], ["\\(","\\)"]]);
          );
          );
          , "mathjax-editing");

          StackExchange.ready(function()
          var channelOptions =
          tags: "".split(" "),
          id: "504"
          ;
          initTagRenderer("".split(" "), "".split(" "), channelOptions);

          StackExchange.using("externalEditor", function()
          // Have to fire editor after snippets, if snippets enabled
          if (StackExchange.settings.snippets.snippetsEnabled)
          StackExchange.using("snippets", function()
          createEditor();
          );

          else
          createEditor();

          );

          function createEditor()
          StackExchange.prepareEditor(
          heartbeatType: 'answer',
          convertImagesToLinks: true,
          noModals: false,
          showLowRepImageUploadWarning: true,
          reputationToPostImages: 10,
          bindNavPrevention: true,
          postfix: "",
          noCode: true, onDemand: true,
          discardSelector: ".discard-answer"
          ,immediatelyShowMarkdownHelp:true
          );



          );













           

          draft saved


          draft discarded


















          StackExchange.ready(
          function ()
          StackExchange.openid.initPostLogin('.new-post-login', 'https%3a%2f%2fmathoverflow.net%2fquestions%2f312236%2fconsecutive-prime-gaps-and-explicit-bound%23new-answer', 'question_page');

          );

          Post as a guest






























          1 Answer
          1






          active

          oldest

          votes








          1 Answer
          1






          active

          oldest

          votes









          active

          oldest

          votes






          active

          oldest

          votes








          up vote
          4
          down vote



          accepted










          The result you quote is due to Baker-Harman-Pintz (2000). I am not aware of any concrete effective version of this result, but if you increase the exponent $0.525$ to $2/3$, then such a variant is available by the work of Dudek. See also my response to this MO question.






          share|cite|improve this answer






















          • Thx, so is there $alpha <1$ such that $cleq 2$ and $n_0$ or $x_0$ is small say less than $10^10$ or $10^12 $ ?
            – Ahmad
            1 hour ago















          up vote
          4
          down vote



          accepted










          The result you quote is due to Baker-Harman-Pintz (2000). I am not aware of any concrete effective version of this result, but if you increase the exponent $0.525$ to $2/3$, then such a variant is available by the work of Dudek. See also my response to this MO question.






          share|cite|improve this answer






















          • Thx, so is there $alpha <1$ such that $cleq 2$ and $n_0$ or $x_0$ is small say less than $10^10$ or $10^12 $ ?
            – Ahmad
            1 hour ago













          up vote
          4
          down vote



          accepted







          up vote
          4
          down vote



          accepted






          The result you quote is due to Baker-Harman-Pintz (2000). I am not aware of any concrete effective version of this result, but if you increase the exponent $0.525$ to $2/3$, then such a variant is available by the work of Dudek. See also my response to this MO question.






          share|cite|improve this answer














          The result you quote is due to Baker-Harman-Pintz (2000). I am not aware of any concrete effective version of this result, but if you increase the exponent $0.525$ to $2/3$, then such a variant is available by the work of Dudek. See also my response to this MO question.







          share|cite|improve this answer














          share|cite|improve this answer



          share|cite|improve this answer








          edited 3 hours ago

























          answered 3 hours ago









          GH from MO

          56.4k5138214




          56.4k5138214











          • Thx, so is there $alpha <1$ such that $cleq 2$ and $n_0$ or $x_0$ is small say less than $10^10$ or $10^12 $ ?
            – Ahmad
            1 hour ago

















          • Thx, so is there $alpha <1$ such that $cleq 2$ and $n_0$ or $x_0$ is small say less than $10^10$ or $10^12 $ ?
            – Ahmad
            1 hour ago
















          Thx, so is there $alpha <1$ such that $cleq 2$ and $n_0$ or $x_0$ is small say less than $10^10$ or $10^12 $ ?
          – Ahmad
          1 hour ago





          Thx, so is there $alpha <1$ such that $cleq 2$ and $n_0$ or $x_0$ is small say less than $10^10$ or $10^12 $ ?
          – Ahmad
          1 hour ago


















           

          draft saved


          draft discarded















































           


          draft saved


          draft discarded














          StackExchange.ready(
          function ()
          StackExchange.openid.initPostLogin('.new-post-login', 'https%3a%2f%2fmathoverflow.net%2fquestions%2f312236%2fconsecutive-prime-gaps-and-explicit-bound%23new-answer', 'question_page');

          );

          Post as a guest













































































          Comments

          Popular posts from this blog

          Long meetings (6-7 hours a day): Being “babysat” by supervisor

          Is the Concept of Multiple Fantasy Races Scientifically Flawed? [closed]

          Confectionery